Sobre el problema relativista de simultaneidad de vagones [cerrado]

Me estoy confundiendo completamente con este problema. La situación es básicamente saber qué eventos tomar en un cuadro para verlos sincronizados en otro. Esbocé la situación así:

vagones

Hay dos vagones A y B de longitud en reposo moviéndose con velocidad v y v visto por un observador C. En el vagón B hay una lámpara que parpadea regularmente.

Supongamos (esta es en realidad la metáfora del problema original) este es un concurso de baile relativista donde C es el juez y los observadores en A y B son una pareja de baile. Lo que quiero saber es cuándo deben bailar los observadores en A y B después de un parpadeo para que el juez C vea su baile sincronizado.

Lo que intento es ver los eventos de su reacción individual de los observadores en el marco de la lámpara y luego transformarlos en el marco del jurado. Entonces, dejando ( t B , X B , 0 , 0 ) ser el evento cuando el bailarn en B reacciona y ( t A , X A , 0 , 0 ) el evento cuando el bailarín en A reacciona visto desde el marco de la lámpara, en el marco del jurado,

t B = γ v ( t B v X B C 2 ) t A = γ v ( t A + v X A C 2 )
que debe ser igual para que C vea estos eventos como simultáneos, entonces (como γ v = γ v )
t B v X B C 2 = t A + v X A C 2
como se ve desde la lámpara, X B = siempre tan
t A = t B v C 2 v X A C 2
y a partir de aquí creo que no sé muy bien cómo proceder; X A debe ser al menos igual a , así que estoy dejando X A = X α para algunos X α > 0 que espero poder determinar, por lo tanto
t A = t B v X α C 2
si dejo X α = 0 , Cómo t A = t B implica t A = t B ? ¿Qué debo hacer con X α ? También estoy pensando en dejar t B = C sería bueno ya que este es el tiempo que tarda la luz en llegar al observador en B, entonces
t A = C v X α C 2
pero, de nuevo, no sé qué hacer con X α . También pensé en tomar tres marcos de referencia, dos para los vagones y uno para C, pero no llegué a ningún lado. También pensé que la adición de velocidad debería interferir con la dilatación del tiempo, pero no veo dónde. Entonces, ¿cómo se debe manejar este problema y usando qué conceptos?

Respuestas (1)

No creo que el ejemplo que usaste sea bueno para ilustrar la relatividad de la simultaneidad. Tiene la complicación adicional de utilizar 3 marcos de referencia, cuando dos deberían ser suficientes. No pude seguir su derivación, pero aquí hay una mucho más simple y más fácil de entender.

Imagine dos observadores, uno a mitad de camino dentro de un vagón de tren a toda velocidad y otro observador parado en una plataforma mientras el tren pasa.
Se emite un destello de luz en el centro del vagón justo cuando los dos observadores se cruzan. El observador a bordo del tren ve la parte delantera y trasera del vagón a distancias fijas de la fuente de luz y, como tal, según este observador, la luz alcanzará la parte delantera y trasera del vagón al mismo tiempo. El observador parado en la plataforma, por otro lado, ve la parte trasera del vagón moviéndose (alcanzándose) hacia el punto en el que se emitió el destello y la parte delantera del vagón alejándose de él. Como la velocidad de la luz es finita y la misma en todas las direcciones para todos los observadores, la luz que se dirige hacia la parte trasera del tren tendrá que cubrir menos distancia que la luz que se dirige hacia el frente. Por lo tanto, los destellos de luz golpearán los extremos del vagón en diferentes momentos.

1) desde el punto de vista de la persona en el tren, la luz llega a ambos extremos simultáneamente ingrese la descripción de la imagen aquíe

2) Desde el punto de vista de la persona en el andén, la luz llega primero a la parte trasera del tren.

ingrese la descripción de la imagen aquí

Gracias por su aclaración allí physics.stackexchange.com/questions/267691/… . Ojalá hubiera visto esa foto antes para hacer mi pregunta más clara. Lo que todavía no entiendo es que en el cuadro 1) el tren se está moviendo en relación con la luz (ya que la velocidad de la luz es constante y no se altera por el marco de referencia), por lo tanto, la luz debería tardar el mismo tiempo en alcanzar la parte delantera en 1) y 2)